Artin Primitive Roots Conjecture (Hooley’s Conditional Proof)

Given a integer a , consider the set S=\{1, a, a^2, \cdots, a^n, \cdots \} . For how many values of prime p , the set exhausts all the non-zero residues. That is for how many primes p , the given integer a primitive element modulo p ?

There are some obvious examples where there are a very few/no such primes. For instance take a=\pm 1 : a=1 only generates one residue class and primitive only for p=2 . a=-1 generates at most two residues classes and is primitive only for p=2, 3 .
If a=n^2 is a perfect square, the residues generated will all be square residues and cannot exhaust (\mathbb Z/p)^{*} . Similar logic applies to perfect powers, but other powers a^k . residues can sometimes exhaust the whole group – for instance if (k, p-1)=1. .

So the squares and -1 . seem to be the only issues and we have the following conjecture:

Conjecture. If a \neq-1 . or a perfect square, then there is a constant C(a)>0 . such that

\displaystyle \left|\left \{p \leq x: a \text{ is a primitive root modulo } p\right\} \right| \sim C(a) \frac{x}{\log x} .

For a=2 ,

\displaystyle C_{\text {Artin }}=\prod_{p \text { prime }}\left(1-\frac{1}{p(p-1)}\right)=0.3739558136 \ldots

That is the set

\displaystyle P(2) =\left\{3,5,11,13,19,29,37,53,59,61,67,83,101,107,131,139,149,163,173,179,181,197, \ldots\right\}

of primes with primitive element 2 has density \approx 0.3739.. among all the primes.

Below we will discuss a conditional proof under GRH. The problem is still open, but the state of the art is Heath-Brown proof that at least one of 2, 3, or 5 is a primitive root modulo infinitely many primes p


Hooley’s Proof conditional on GRH:

Let us fix a=2. Now 2 is a primitive root modulo p iff

\displaystyle 2^{d} \not \equiv 1 \mod p for d|p-1, d\neq p-1. That is we dont want

\displaystyle 2^{\frac{p-1}{q}} \equiv 1 \mod p to hold for any prime q.

We first estimate S_1 the set of primes p for which there are no obstructions like above for q < z. We also estimate S_2, the primes for which there is an obstruction z<q<x, then our required count for primes p where 2 is primitive is given |S_1| +O(|S_2|) because those elements of S_1 that shouldn’t be included in our count are all elements of S_2.

By inclusion-exclusion we write S_1 as sifted sum where we remove primes for which the obstruction holds at different primes q

\displaystyle |S_1| =\sum_{d|(n, \displaystyle\prod_{q<y})} \mu(d) \left|\{p \le x: q| p-1, 2^{\frac{p-1}{q}} \equiv 1 \mod p \text{ for all  } q|d \}\right|

Now each of the term inside can be seen as the number of splitting primes in the compositum of Kummer extensions \displaystyle K_{q}=\mathbb{Q}\left(2^{1 / q}, \zeta_{q}\right) for q|d . So by GRH (for the Dedekind zeta function of this compositum), we get the count split primes

\displaystyle =\frac{1}{d\varphi(d)} \text{Li}(x)+O\left(x^{1/2} \log (d x)\right)

Plugging this in we get

\displaystyle |S_1| = \sum_{d|(n, \displaystyle \prod_{q<y} } \mu(d) \left(\frac{\text{Li}(x)}{d \varphi(d)}+O\left(x^{1 / 2} \log (d x)\right)\right)

\displaystyle  \sim C(2) \frac{x}{\log x}

by taking y \le \log x and observing that

\displaystyle =\sum_{d | \displaystyle \prod_{q<y} } \frac{1}{d \varphi(d)}=\prod_{q <y}\left(1-\frac{1}{q(q-1)}\right) \sim C(2)=\prod_{q}\left(1-\frac{1}{q(q-1)}\right) .

To estimate S_2 the primes that have a obstruction z<q<x, we split the range of primes into three parts small, medium and large (C\log x \le q \le \frac{\sqrt x}{\log x^2}, \frac{\sqrt x}{\log^2 x} \le q \le \sqrt x \log x, and \sqrt x \log x \le x \le x

Again for each of the primes q, these counts are exactly split primes in the Kummer extension K_q.

For small primes, summing over all the q gives a bound \ll \frac{x}{(\log x)^{2}}.

For medium primes we just use the condition p =1 \mod q and estimate the primes in arithmetic progressions by Brun-Titchmarsh

\displaystyle \pi(x ; q, 1) \leq 2 \frac{x}{\varphi(q) \log (x / q)}

gives an admissible bound like \ll \frac{x \log \log x}{(\log x)^{2}} .

Finally for the large primes, use the fact that \displaystyle 2^{\frac{p-1}{q}} \equiv 1 \mod p, to bound the number of primes p by the number of prime factors of all the numbers \displaystyle 2^n-1, where \displaystyle n =\frac{p-1}{q} \le M=\frac{\sqrt x}{\log x}

The number of prime factors of \displaystyle \prod_{n \le M} (2^M-1) is small by simple estimates like \omega(2^n-1) \le \log_2 n .

Putting all these together we get the desired estimate ~C_2 \frac{x}{\log x}.


C. Hooley, On Artin’s conjecture, J. reine angew. Math. 225 (1967), 209-220.

D. R. Heath-Brown, Artin’s conjecture for primitive roots, Quart. J. Math. Oxford Ser. (2) {3 7}(1986), 27-38.

Posted in $.

One thought on “Artin Primitive Roots Conjecture (Hooley’s Conditional Proof)

Leave a comment